Psych EOR

Pataasin ang iyong marka sa homework at exams ngayon gamit ang Quizwiz!

84) A 27-year-old man is brought to the emergency department by his wife. She says that he has been "acting crazy" for the last 2 weeks. He has hardly slept for the past 7 days and instead has worked on miscellaneous projects around the house. The patient spent several thousand dollars on new power tools to accomplish these tasks. When questioned, his speech is pressured. He feels "spectacular" and is creating an "architectural masterpiece." The patient has had 2 previous depressive episodes. Which of the following medications is the most appropriate agent for long-term management of this patient? a. Buproprion b. Chlorpromazine c. Haloperidol d. Paroxetine e. Valproate

E 84. Diagnosis = acute manic episode (bipolar I); preferred long-term maintenance therapy = lithium, valproate, lamotrigine, quetiapine

30) A 32-year-old man comes to the office due to extreme nervousness, irritability, restlessness, muscle tension, and insomnia for the past 6 months. The patient fears making mistakes in his job as an attorney and often worries that he has said or done something wrong. His worrying makes it difficult to concentrate or perform his duties efficiently. The patient is also engaged to be married and spends sleepless nights thinking about the responsibilities of married life. He worries that he will not be a good husband or father. Which of the following medications is most appropriate for this patient? a. Buproprion b. Diazepam c. Propranolol d. Quetiapine e. Venlafaxine

E 30. Diagnosis = generalized anxiety disorder; treatment = SSRI or SNRI (venlafaxine) • Buproprion not effective in GAD; may worsen insomnia/anxiety

23) A 45-year-old woman returns to the office for follow-up of depression. The patient was diagnosed with major depressive disorder 6 weeks ago and was prescribed sertraline. She reports some improvement in depressive symptoms but now has sexual dysfunction with decreased libido and anorgasmia. Which of the following is the best next step in the management of this patient? a. Add buproprion and taper sertraline b. Add venlafaxine and taper sertraline c. Decrease dose of sertraline d. Discontinue sertraline and switch to buproprion e. Discontinue sertraline and switch to citalopram

A 23. Treatment of SSRI-related sexual dysfunction: • Rule out other causes (depression, medical conditions, primary sexual disorder, relationship issues, stress, substance abuse) • SSRI dose reduction (for patients on high-dose SSRI), switch to a non-SSRI antidepressant, or add buproprion/sildenafil to current SSRI

105) A 42-year-old woman is brought to the emergency department by her husband for new-onset tremors and extreme restlessness. She feels nauseated and anxious. The patient recently injured her back and was prescribed tramadol. She has major depression with psychotic features for which she takes venlafaxine and aripiprazole. Temperature is 101º F, BP is 160/100, pulse is 125, and RR is 20. On exam, the patient is flushed and diaphoretic and her voice is tremulous. Mild rigidity and tremors are noted in her lower extremities. Deep tendon reflexes are 3+. Pupillary dilation and ocular clonus are present. Dysregulation of which of the following neurotransmitters is the most likely cause of this patient's symptoms? a. Acetylcholine b. Dopamine c. Gamma-aminobutyric acid d. Norepinephrine e. Serotonin

E 105. Diagnosis = serotonin syndrome (caused by combo of venlafaxine and tramadol); triad = AMS, autonomic dysregulation, neuromuscular hyperactivity

102) A 54-year-old man comes to the office due to concerns about having pancreatic cancer after a coworker died of the disease 6 months ago. The patient has no epigastric pain, jaundice, or weight loss. However, he worries constantly because, in researching the illness, he read that it may not have obvious symptoms in the early stages and can be rapidly fatal. The patient saw another health care provider 2 months ago, and physical exam, laboratory evaluation, and abdominal CT scan were normal. However, the patient has noticed occasional stomach noises after eating and would like to have additional testing done. Which of the following is the most likely diagnosis in this patient? a. Conversion disorder b. Factitious disorder c. Generalized anxiety disorder d. Illness anxiety disorder e. Somatic symptom disorder

D 102. Features of illness anxiety disorder: preoccupation with the idea of having a serious illness, but patient has minimal to no symptoms.

101) A 64-year-old woman is brought to the emergency department by her husband and a friend. The patient was diagnosed with aplastic anemia a year ago. She has responded poorly to immunosuppressive therapy and has required multiple blood transfusions. The patient is currently febrile, disoriented, and unable to respond to questions. Complete blood count shows pancytopenia. During the evaluation, the friend presents a notarized health care proxy document signed a month ago in which the patient designated her friend as her proxy. The friend tells the treating health care provider that the patient did not want to undergo any further blood transfusions. The husband adamantly insists that the patient receive a blood transfusion and says that it is what she would have wanted. Who should make the decision regarding the transfusion? a. Courts b. Ethics committee c. Friend d. Husband e. Treating healthcare provider

C 101. The patient's health care proxy (the friend) is legally obligated to make decisions in accordance with the patient's wishes when the patient becomes incapacitated/lacks decision-making capacity. Family members (the spouse) can only make decisions if the patient has not appointed another health care proxy. The health care proxy overrules all other decision-makers.

106) A 72-year-old woman is admitted to the hospital because of worsening suicidal ideation for the past month. The patient has been refusing meals in order to "sacrifice" herself for "the greater good." She has stopped communicating with her family and friends and spends most of her days sleeping or staring at the television. The patient has been treated with multiple antidepressants in the past for recurrent major depressive disorder. Two weeks ago, she discontinued taking venlafaxine because she felt it "interfered" with her plan to kill herself. BMI is 16.5. Physical exam shows decreased skin turgor and dry mucous membranes. Which of the following is the best next step in management? a. Clozapine b. Cognitive behavioral therapy c. Electroconvulsive therapy d. Imipramine e. Olanzapine

C 106. Diagnosis = major depressive disorder with psychotic features; because this case is emergent (patient is suicidal, refusing to eat, and has not become dehydrated), she should be treated with ECT, which has a faster onset than a combo of antidepressant + antipsychotic

107) A 32-year-old woman comes to the office due to depressed mood, fatigue, irritability, and feelings of low self-worth on more days than not for the past 2 years. The patient reports that her symptoms are worse during the week prior to menstruation but occur at other times as well. During the week prior to menses, she experiences physical symptoms of mild bloating and breast tenderness that resolve with the onset of menses. Which of the following is the most likely diagnosis in this patient? a. Borderline personality disorder b. Dysmenorrhea c. Persistent depressive disorder (dysthymia) d. Premenstrual dysphoria disorder e. Premenstrual syndrome

C 107. Features of dysthymia (persistent depressive disorder): • Chronic depressive symptoms for ≥ 2 years • Distinguished from PMS/PMDD by occurring throughout the menstrual cycle • Any symptom-free periods are < 2 months long; patient has symptoms for most of the days of the past 2+ years

109) A 9-year-old girl is brought to the office by her mother for assessment of "bad stomach aches." The patient has missed 5 days of school in the past few weeks due to intermittent abdominal pain that is "achy all over" and worse in the morning. The girl used to be talkative and outgoing, but now does not visit her friends and instead takes naps after school. The patient has been distracted in class lately and less engaged in group activities. Her father lost his job 3 months ago and has been home more often since then. When interviewed, the patient is withdrawn and makes poor eye contact. When asked if she feels depressed, she says, "Everything will be better soon since no one will have to take care of me anymore." She avoids answering further questions. Which of the following is the best next step in the management of this patient? a. Arrange for an in-home social work assessment b. Contact the patient's school for further information c. Obtain an emergent psychiatric assessment d. Recommend antidepressant treatment e. Recommend family therapy

C 109. Depression in childhood = often has somatic symptoms; patient stating "no one will have to take care of me anymore" should be concerning for suicidal ideation = emergency

108) A 29-year-old woman comes to the office due to persistent fatigue over the last 4 years. She has also felt unhappy during this period, ever since being let go from her previous job. The patient says she has "little energy to do things." When asked what she enjoys, she replies that "everything in life is a chore" and that she feels hopeless that her life will improve. The patient has no suicidal thoughts, problems with concentration, or changes in appetite or sleeping patterns. She drinks 1 or 2 glasses of wine on weekends. Detailed workup, including urine toxicology screen, is negative. Which of the following is the most likely diagnosis in this patient? a. Adjustment disorder b. Borderline personality disorder c. Major depressive disorder d. Persistent depressive disorder (dysthymia) e. Substance-induced mood disorder

D (See 107)

104) A 25-year-old man is brought to the emergency department due to severe agitation and aggressive behavior. He speaks loudly and rapidly, stating that he has "superpowers" of mind control and that violence is the only way to defend against "the conspiracy." The patient has bipolar disorder and is prescribed olanzapine, lithium, and fluoxetine, although it is unclear if he has been compliant. The patient requires several doses of medication in the emergency department to calm down and is admitted to the hospital for further psychiatric evaluation. The following night he is found lying very still on his bed and does not respond to questions. Temperature is 105.1º F, BP is 157/90, pulse is 102, and RR is 20. Exam is significant for diaphoretic and diffuse rigidity in the upper and lower extremities. Which of the following is the most likely explanation for this patient's current symptoms? a. Acute dystonic reaction b. Drug-induced Parkinsonism c. Lithium toxicity d. Neuroleptic malignant syndrome e. Serotonin syndrome

D 104. Diagnosis: manic episode with psychotic features - patient has received multiple doses of antipsychotics, causing neuroleptic malignant syndrome • Hyperthermia • Sympathetic hyperactivity (diaphoresis, tachycardia) • muscle rigidity • AMS

103) A 28-year-old man is brought to the emergency department by his roommate 45 minutes after he was observed having a seizure. The patient has no known history of a seizure disorder. The roommate reports that the patient has been depressed since a serious motorcycle collision 5 years ago and recently completed a drug rehabilitation program for cocaine and prescription opioid abuse. He thinks the patient is taking an antidepressant but is not sure of which one. On exam, the patient is sedated, disoriented, and flushed. Temperature is 101º F, BP is 90/50, and pulse is 130. Pupils are dilated and bowel sounds are decreased. ECG shows sinus tachycardia and QRS duration of 130 msec. Which of the following is the most likely cause of this patient's symptoms? a. Buproprion overdose b. Cocaine overdose c. Opioid withdrawal d. Serotonin syndrome e. Trycyclic antidepressant overdose

E 103. Features in this patient consistent with TCA overdose: seizure, signs of anticholinergic toxicity (mydriasis, hyperthermia, flushed skin, decreased bowel sounds), long QRS (> 100 msec) • Other signs/symptoms: AMS, tachycardia, HoTN • Treat prolonged QRS with sodium bicarbonate to reduce risk of seizures/ventricular arrhythmia

58) A 34-year-old woman comes to the office with her husband due to behavioral changes over the past 6 weeks. The husband says, "She's not an angry person, but ever since she was in a car accident, little things seem to set her off. She yells and honks at people for for not using their turn signals and gets upset when we have to wait for a table at restaurants." The patient describes difficulty sleeping due to thoughts "swirling" in her head and feeling panicked every time she wakes up. She sustained a wrist fracture in a motor vehicle collision 2 months ago; medical history is otherwise unremarkable. Vital signs and physical exam are normal. On mental status exam, she appears restless and tired. Which of the following is the best next step in management of this patient? a. Begin buspirone b. Obtain MRI of the head c. Recommend cognitive-behavioral therapy d. Refer for group therapy e. Start lithium

C 58. Diagnosis = PTSD; 1st-line treatment = CBT

85) A 65-year-old man is brought to the office by his daughter due to decreased activity and impaired sleep over the past month. He was diagnosed with Parkinson's disease 2 years ago. According to his daughter, he displays very few emotions and does not want to do anything. He just sits in his chair and watches TV with a blank expression, and does not want to leave his house. He has been waking up early in the morning, which is very unusual for him. The patient is adherent with his levodopa/carbidopa treatment. Physical exam shows mask-like facies and slowed movements. Mild hand tremor and postural instability have improved since last visit. Which of the following is the most appropriate next step in pharmacological management of this patient? a. Add amantadine b. Add quetiapine c. Add sertraline d. Increase the dose of levodopa/carbidopa e. Switch to selegiline

C 85. Diagnosis = acute depressive episode; depression affects up to 50% of patients with Parkinson's; treat with antidepressants (e.g. SSRI)

18) A 15-year-old girl is brought to the clinic by her mother, who became concerned after hearing her daughter vomit in the bathroom on 2 separate occasions last week. The patient is preparing for a debate competition and says that she occasionally vomits from nervousness. She describes intense pressure to perform well in school and is very self-conscious. Her last menstrual period was 10 weeks ago. BMI is at <5th percentile. Physical exam shows a thin girl with dental caries, bilateral parotid gland enlargement, and a scaphoid abdomen. Which of the following is the most likely diagnosis? a. Anorexia nervosa b. Body dysmorphic disorder c. Bulimia nervosa d. Generalized anxiety disorder e. Performance anxiety with panic attacks

A 18. Low body weight/BMI and physical signs consistent with self-induced vomiting = binge/purge subtype of anorexia nervosa (bulimia nervosa would have a normal body weight)

21) A 28-year-old graduate student comes to the office due to anxiety and trouble concentrating. He has worked part-time as a volunteer firefighter and reports that he has been struggling to concentrate ever since witnessing a child's death in a house fire 3 weeks ago. On two occasions the patient began to sweat, shake, and had to run out of the classroom because he started to have shortness of breath. He also reports having trouble sleeping and being irritable with friends and family. The patient says, "Sometimes I'll be sitting in class, and I suddenly feel like I am back in the burning house. I can still see the child." He has missed almost half of his classes to avoid being in crowded classrooms and recently quit his volunteer job. Which of the following is the most likely diagnosis? a. Acute stress disorder b. Adjustment disorder c. Depersonalization/derealization disorder d. Panic disorder e. Post-traumatic stress disorder

A 21. Acute stress disorder = symptoms similar to PTSD but lasting ≥ 3 days and ≤ 1 month (when symptoms persist for ≥ 1 month, the diagnosis is PTSD)

24) A 14-year-old girl comes into the office for follow-up of asthma. After a review of the patient's respiratory symptoms, which have remained stable on her current medication regimen, she hesitantly says that she wishes she were a boy and that she has felt this way "my whole life." She has not told anyone due to fear of disapproval, saying, "My parents will kill me if they find out." She has tried to manage her feelings on her own but is becoming more distressed as pubertal changes occur, with increased breast development and hair growth. She asks for advice about her situation. In addition to providing support, which of the following is the most appropriate? a. Ask if the patient is threatened either at home or at school b. Inform the parents so they can decide about how to proceed c. Psychotherapy referral to discuss gender conformity d. Reevaluate in a few years because gender identity can change e. Start hormonal treatment to delay puberty

A 24. Initial steps for adolescent patient with gender dysphoria: nonjudgmental support, express interest in understanding patient's experience, and determine whether there is risk of harm

35) A 21-year-old man comes to the emergency department due to a broken nose. He got into a fight at a bar after he became angry with another man for looking at his girlfriend. The patient is not allowed back at that bar because he has started so many fights there. He says, "The other guy looks much worse than I do, but he had it coming." The patient has had multiple jobs in the past year and was recently fired for repeated absences. He states, "That job was beneath me. I can make more money doing something else." Starting in middle school, the patient had repeated suspensions for truancy and fighting; he blames his failure to be accepted into college on the teachers who reported his behavior. Which of the following is the most likely diagnosis in this patient? a. Antisocial personality disorder b. Borderline personality disorder c. Conduct disorder d. Intermittent explosive disorder e. Narcissistic personality disorder

A 35. Features of antisocial personality disorder: • Disregard for the safety of self/others • Unstable employment history • Lack of remorse • Evidence of conduct disorder before age 15

44) A 28-year-old woman is brought to the office by her husband, who is concerned about her recent behavior. For the past 6 months, the patient has reused to eat any food that is not prepackaged out of fear of becoming ill. Before eating, she carefully examines the food on her plate, checking for any possible contaminants. The patient believes that someone has been poisoning her food, which has caused her to feel more fatigued than usual. She has continued to work but seldom interacts with coworkers. The patient has no psychiatric history. Exam shows a 2.27 kg (5 lb) weight loss since a prior visit 9 months ago. The patient's mood is anxious, and her affect is tense. Which of the following is the most likely diagnosis in this patient? a. Delusional disorder b. Illness anxiety disorder c. Obsessive compulsive disorder d. Paranoid personality disorder e. Schizophrenia

A 44. Features of delusional disorder: • ≥ 1 delusion for ≥ 1 month • No other prominent psychotic symptoms • Behavior not obviously odd/bizarre; patient still able to function • Treatment = antipsychotics, CBT

55) A 32-year-old woman comes to the office because she has felt sad and worthless for the past 3 months. The patient has difficulty sleeping, decreased appetite, and no longer enjoys spending time with friends. The patient is started on first-line pharmacological treatment. Two days later, the patient is brought to the emergency department after being found lying down next to an empty bottle of the prescribed medication. Temperature is 102º F, BP is 146/92, and pulse is 118. The patient is disoriented, tremulous, and diaphoretic. She has abdominal cramps and diarrhea. Bilateral lower extremities have hyperreflexia and inducible ankle clonus. The patient is admitted to the hospital for supportive therapy, but she remains disoriented with only limited improvement. Which of the following is the antidote for this patient's condition? a. Cyproheptadine b. Flumazenil c. Haloperidol d. Naloxone e. Propanolol

A 55. Diagnosis = serotonin syndrome (AMS, autonomic hyperactivity, and neuromuscular excitation in the setting of SSRI overdose); treatment = cyproheptadine (serotonin antagonist)

59) A 13-year-old girl is brought to the office by her mother due to concerns about the girl's adjustment to a new school. The patient has always been shy and somewhat awkward. The new school emphasizes group projects, which the mother thought would help her daughter be more social, but has instead has made her extremely anxious. The patient is bright but underachieves at school and has few friends. She has an obsessive interest in horses. During the interview, the patient makes poor eye contact, interrupts questions by changing topics, and mentions facts about different horse breeds. When her vital signs are checked, she touches the blood pressure cuff repeatedly. Which of the following is the most likely explanation for this patient's behavior? a. Autism spectrum disorder b. Obsessive-compulsive disorder c. Schizoid personality disorder d. Schizotypal personality disorder e. Social anxiety disorder

A 59. Features of this patient consistent with ASD: social awkwardness, intense interest in/restricted focus on a single topic, unusual sensory interest; diagnosis may be made later in childhood in high-functioning autism

69) A 19-year-old woman is brought to the emergency department due to bizarre behavior. Her father found the patient this morning sitting up in bed, staring at the wall. The patient did not respond to questions and was very stiff when her father tried to help her stand. The patient has bipolar disorder and takes lithium, risperidone, and fluoxetine; her dosages of risperidone and fluoxetine were both increased recently due to residual depressive symptoms. Vital signs are normal. When asked what today's date is, the patient repeats, "Date, date, date," repeatedly. She resists the examiner's movement of her arms, increasing her resistance in proportion to the applied force. Which of the following is the most likely diagnosis for this patient? a. Catatonia b. Delirium c. Drug-induced Parkinsonism d. Neuroleptic malignant syndrome e. Serotonin syndrome

A 69. Features of catatonia: immobility, mutism, negativism (resistance to passive movement), echolalia (repeating others' words); catatonia most often develops in the context of a mood disorder (e.g. bipolar) but can also develop in psychotic disorders, ASD, or general medical illness

72) A 37-year-old woman is brought to the hospital after she was found wandering in the street. The patient is unable to provide a coherent history of recent events but mentions that she had a psychiatric hospitalization in her 20s. The patient is disheveled and malodorous. On mental status exam, she is alert and smiles pleasantly, but it is difficult to hold her attention. The patient gets up from her chair several times to stare at the air conditioning vent and to put her ear against the wall, as if listening for something. When asked about her behavior, she says, "Jingle jangle, jingle jangle." The patient is admitted to the psychiatric unit. Which of the following is the best initial treatment for this patient? a. Aripiprazole b. Clozapine c. Haloperidol d. Lithium e. Lorazepam

A 72. Diagnosis = acute psychosis most likely due to schizophrenia; initial treatment = 2nd-gen antipsychotics (e.g. aripiprazole); clozapine is a 2nd-gen antipsychotic but is not used 1st-line due to risk of agranulocytosis (used only after failure of 2 other antipsychotics)

77) A 53-year-old man has been prescribed chlorpromazine for many years for schizophrenia. His psychotic symptoms have been stable, but he has developed a tremor in his hands and he walks slower. His other medical conditions include obesity and hypercholesterolemia. The patient has been trying to exercise and has managed to lose 5 kg (11 lb) since his last visit. He is concerned about his hands shaking and would like to lose more weight. BMI is 37. The patient has psychomotor retardation, slow speech, and lack of facial expressions. A resting tremor of the hands is noted. He would like to switch to another medication. Which of the following medications would be most appropriate for this patient? a. Aripiprazole b. Clozapine c. Haloperidol d. Lamotrigine e. Olanzapine

A 77. Diagnosis = extrapyramidal side effects (Parkinsonism) caused by 1st-gen antipsychotic (chlorpromazine); treatment = switch to 2nd-gen antipsychotic (aripiprazole) • Aripiprazole preferred over olanzapine/clozapine because of lower potential for causing weight gain/metabolic syndrome

86) A 9-year-old girl is brought to the office for behavioral problems at school. The patient was sent home a week ago after she threw a pencil and cried when the teacher criticized her handwriting. Since then, she has had stomachaches and headaches every morning and asks to stay home. Over the past year, the patient has become increasingly self-conscious and cannot relax because she thinks that she has said something wrong and that the other kids are laughing at her. Several times a week, she struggles to fall asleep due to concerns that someone might break into the house and insists that her mother check the locks. Which of the following is the most likely diagnosis in this patient? a. Generalized anxiety disorder b. Obsessive-compulsive disorder c. Obsessive-compulsive personality disorder d. Separation anxiety disorder e. Social anxiety disorder

A 86. Features of this patient consistent with generalized anxiety disorder: • Excessive concern about multiple issues • Irritability • Difficulty falling asleep • Poor concentration • (GAD in children can manifest as a need for perfection/order)

88) A woman brings her 19-year-old daughter to the office due to concerns that she is hypersensitive, depressed, and socially isolated. The mother worries that her daughter is typically withdrawn, has few close friends, and does not date. The girl is also moody and tends to overreact to the slightest criticism. The patient describes her mood as "unhappy" but has had no change in sleep or appetite. She works as a filing clerk and recently turned down a higher-paying job because she prefers to work alone. When questioned about her social isolation, she says, "Who would want to be friends with someone ugly and stupid like me anyway?" Which of the following is the most likely explanation for this patient's behavior? a. Avoidant personality disorder b. Borderline personality disorder c. Dependent personality disorder d. Persistent depressive disorder (dysthymia) e. Schizoid personality disorder

A 88. Features of avoidant personality disorder (cluster C): avoids social interaction due to fear of rejection and hypersensitivity to criticism, struggles with feelings of inadequacy, can have occupational dysfunction due to difficulty interacting with coworkers

17) A 15-year-old boy is brought to the office after he was caught shoplifting from an art supply store. For the past 2 weeks, the patient has become loud and rude. He has stayed up multiple nights in a row, drawing in his room. The patient says that he lives in "the most boring place in the world" and that his parents are jealous he is "making a masterpiece that will be world famous." On mental status exam, he is easily distracted and struggles to sit still. The patient speaks quickly and becomes irritable when attempts are made to clarify what he says. Urine drug screen is negative. Which of the following is the most likely explanation for this patient's behavior? a. Attention deficit hyperactivity disorder b. Bipolar disorder c. Conduct disorder d. Normal adolescent behavior e. Oppositional defiant disorder

B 17. Abrupt change in behavior, decreased need for sleep, goal-directed activity, grandiosity, and risky behavior = manic episode; bipolar disorder requires only 1 manic episode for diagnosis

99) A 21-year-old college student is brought to the emergency department by ambulance after a seizure. The patient swallowed an unknown substance at a concert festival. Soon afterward, he appeared to be interacting with others. A few hours later, the patient became disoriented, appeared flushed, started sweating profusely, and then had a seizure. He has depression, for which he takes a selective serotonin reuptake inhibitor. Temperature is 104º F, BP is 170/96, pulse is 120, and RR is 18. On exam, the patient is flushed, diaphoretic, and hyperreflexive with occasional myoclonus and continues to be disoriented. Pupils are dilated with minimal reaction to light. Serum sodium level is 121. The patient most likely used which of the following substances? a. Benzodiazepines b. Ecstasy c. Ketamine d. Opioid narcotics e. Phencyclidine

B 99. Features of ecstasy (MDMA) abuse: • Euphoria • Tachycardia/HTN • Hyperthermia • Serotonin syndrome • Hyponatremia • Seizures • Mydriasis

19) A 28-year-old man comes to the office because he is "not feeling like himself." The change began soon after his older brother died of cancer 3 months ago. The patient feels sad and has lost interest in work and going out with friends on weekends. He often awakens very early in the morning and is unable to fall back asleep. The patient feels "very down," especially when he first wakes up, but says that his mood rarely improves during the day. His poor energy and concentration have started to affect his performance at work. He has been eating less and has lost 5.4 kg (12 lb) over the past few weeks. The patient has no psychiatric history. Which of the following is the most likely diagnosis? a. Acute stress disorder b. Major depressive disorder c. Normal grief reaction d. Persistent depressive disorder (dysthymia) e. Persistent complex bereavement disorder

B 19. Patient's symptoms are sufficient in number, duration (≥ 2 weeks), and severity to meet the diagnostic criteria for major depressive episode; ≥ 1 major depressive episode with no history of mania/hypomania = major depressive disorder • Persistent complex bereavement disorder (complicated grief): maladaptive, ruminative thoughts and dysfunctional behaviors related to persistent yearning for the deceased. (This patient's symptoms do not revolve around yearning for his brother)

20) An 18-year-old boy is brought to the emergency department by his mother due to his strange behavior. The mother says that he laughs inappropriately and seems sluggish. The boy has recently been hanging out with a new group of friends and has also stopped attending family events. He appears apathetic about his schoolwork. When asked in private, the boy admits to smoking marijuana. The patient most likely has which of the following examination findings? a. Bradycardia b. Conjunctival injection c. Miosis d. Nystagmus e. Respiratory depression

B 20. Marijuana intoxication: mild euphoria, inappropriate laughter, sedation, slowed reflexes, impaired motor coordination, distorted sensory perceptions, cognitive impairment, conjunctival injection, tachycardia, increased appetite, dry mouth

27) A 7-year-old girl is brought to the office by her mother. Over the past month, the girl has awakened almost every night, come into her parents' room, and insisted on sleeping the remainder of the night in their bed. The girl wakes abruptly in a panicky sweat, crying and screaming, "I'm so scared; monsters are chasing me." She breathes rapidly and shakes but is able to calm down after being consoled by her parents and hugging a favorite stuffed animal. The girl also started a new school 1 year ago. Although the girl is shy, she enjoys school and has several good friends. Which of the following is the most likely diagnosis in this patient? a. Adjustment disorder b. Nightmare disorder c. Nocturnal panic attack d. Separation anxiety disorder e. Sleep terror disorder

B 27. Nightmare disorder: recall of the dream content, child fully alert upon awakening and is consolable (nightmares occur during REM sleep and are more frequent during the second half of the night) • Sleep terrors: more common in the first 3rd of the night (non-REM sleep)

40) A 28-year-old woman comes to the office for evaluation of weight gain. The patient constantly worries that her "huge stomach" protrudes and that she cannot lose weight in her midsection despite following a low-calorie, high-protein diet and performing abdominal exercises daily. She spends multiple hours a day reading posts online about exercising and eating. BMI is 21. The abdomen is soft and flat with normoactive bowel sounds and no tenderness to palpation. The patient denies depression but states that she feels anxious most of the time and avoids socializing because "everyone stares at my stomach." Which of the following is the most likely diagnosis in this patient? a. Anorexia nervosa b. Body dysmorphic disorder c. Bulimia nervosa d. Generalized anxiety disorder e. Obsessive-compulsive disorder

B 40. Features of body dysmorphic disorder: • Preoccupation with ≥ 1 perceived physical defects • Defects not readily apparent to others or nonexistent • Repetitive behavior/ideation ins response to preoccupation • Significant distress/impairment

48) A 53-year-old man comes to the office due to persistent fatigue. The patient started a therapeutic dose of fluoxetine 2 months ago after being diagnosed with major depressive disorder. He says, "I still feel like I'm fatigued every day and have to force myself to go to work in the mornings. I haven't even wanted to have sex with my wife. It's been going on for almost a month, and it's really bothering me." The patient has had a 4 kg (9 lb) weight gain over the past 2 months. Vital signs and physical exam are normal. He appears depressed and speaks softly throughout the interview. Which of the following is the next best step in the management of this patient? a. Continue fluoxetine and add sildenafil b. Discontinue fluoxetine and begin buproprion c. Discontinue fluoxetine and begin mirtazepine d. Discontinue fluoxetine and begin venlafaxine e. Increase fluoxetine dose

B 48. Patient with no significant response after ≥ 6 weeks of SSRI and/or SSRI-related sexual dysfunction: discontinue SSRI and start buproprion (norepinephrine-dopamine reuptake inhibitor; does not cause sexual dysfunction)

49) A 55-year-old woman comes to the office due to insomnia and fatigue that began shortly after her divorce was finalized a year ago. The patient used to sleep 7-8 hours without difficulty; however, over the past year she has had increasing difficulty falling asleep and started drinking 2-3 glasses of wine before bedtime to help. Despite falling asleep more quickly, she has recently started to regularly wake up around 3:00 AM. Her symptoms have progressively worsened and she now lies awake for several hours in the middle of the night, but she rarely experiences anxiety during the day. She has recently started showing up late for work. Vital signs are normal. Physical exam shows a mild tremor, but no other abnormalities. Lab results are as follows: Hematocrit: 34%, MCV: 106, AST: 85, ALT: 42. Which of the following is the most likely diagnosis in this patient? a. Adjustment disorder b. Alcohol use disorder c. Generalized anxiety disorder d. Insomnia disorder e. Persistent depressive disorder

B 49. Features of this patient consistent with alcohol use disorder: • ≥ 14 drinks/week • AST:ALT ratio ≥ 2:1 (patient's = 85:42) • Macrocytosis (patient's MCV = 106) • Alcohol tolerance (decreasing sedative effects over time) • Impaired functioning (work tardiness)

51) A 35-year-old man returns to the office for his third visit in 6 months because he is convinced that he is infected with parasites. He has gained 1.4 kg (3 lb) since his last visit 2 months ago and believes that his abdomen is distended due to growing parasites. The patient believes that he became infected with a tapeworm while on a business trip to Asia 6 years ago. Since then, he has seen multiple health care providers for this concern, with no evidence of infection. He has had several negative stool examinations that he claims are false. The patient spends hours researching parasitic diseases. Physical exam is normal. The patient's mood is anxious and his affect is tense. He has no auditory hallucinations but describes a sensation of parasites crawling under his skin. Which of the following is the most likely diagnosis? a. Body dysmorphic disorder b. Delusional disorder c. Generalized anxiety disorder d. Obsessive-compulsive disorder e. Somatic symptom disorder

B 51. Features of this patient consistent with delusional disorder: persistent, false, fixed belief that he has parasites despite multiple negative tests; somatic delusions typically result in numerous health care visits and refusal to accept negative test results

6) A 40-year-old woman comes to the emergency department fearing she is having a heart attack. While clutching her chest and breathing heavily, she says "I feel like I'm dying." BP is 125/86 and pulse is 110 and regular. Laboratory evaluation and ECG show no abnormalities. The pain resolves within 10 minutes without treatment, and the patient reports, "I was taking the bus home from work when my chest started feeling really tight. I'm lucky my friend was there and able to help me get to the hospital. What if she's not there next time?" She describes experiencing similar episodes that are characterized by a pounding heart, trembling, dizziness, and sweating. This patient most likely has which of the following additional disorders? a. Acute stress disorder b. Agoraphobia c. Brief psychotic disorder d. Dependent personality disorder e. Separation anxiety disorder

B 6. Patient's diagnosis = panic disorder; patients with panic disorder often develop agoraphobia (avoidance of situations in which they may feel trapped/helpless)

68) A 41-year-old woman and her 19-year-old daughter come to the emergency department out of concern that they have been poisoned. The mother reports that their landlord is harassing them because they have been late on their last few rent payments. She says, "He turns down the heat to make us freeze and asks the other tenants to spy on us. Last month, he installed video cameras in our living room while we were out." Today, the mother tasted something odd in her food and is convinced that the landlord poisoned them. She came immediately to the hospital for treatment and to obtain evidence of the poisoning. The daughter agrees with her mother's account, adding that she thinks she saw the landlord lurking outside their apartment the night before. Laboratory testing for both is unremarkable. Which of the following is the most appropriate course of action? a. Admit the mother and daughter to different psychiatric units b. Interview the daughter alone c. Obtain collateral information from the landlord d. Provide reassurance and schedule outpatient follow-up for them both e. Recommend a trial of antipsychotics for both the mother and daughter

B 68. Diagnosis = shared psychotic disorder (same delusion present in individuals who share a close relationship); most important 1st step = separate the patients

75) A 27-year-old woman is brought to the emergency department by police after she became tearful and distraught when the officers refused to initiate an investigation of her former boyfriend. The patient recently broke up with the boyfriend. One week ago, she was fired from her job for frequent lateness. Since then, she has slept poorly and has become convinced that her ex-boyfriend plotted with her former supervisor to get her fired. She says she heard them whispering about her outside her apartment at night. The patient is admitted to the hospital and is discharged after several days with no medication. At a 2-week follow-up visit, she exhibits no paranoid beliefs and reports that she has started looking for a new job. Which of the following is the most likely diagnosis? a. Borderline personality disorder b. Brief psychotic disorder c. Delusional disorder d. Paranoid personality disorder e. Schizophreniform disorder

B 75. Features of brief psychotic disorder: ≥ 1 day but < 1 month, sudden onset and full return to function

78) A 32-year-old man is brought to the office for an evaluation after he was placed on probation at work for frequent absences and arguing with his boss. For the past 9 months, he has had unpredictable mood swings with irritability, aggressive behavior, and paranoia about his coworkers stealing his job. At other times, he is quiet and withdrawn. The patient is cooperative but tense, irritable, and talkative. He is physically restless, mildly tremulous, diaphoretic, and hypervigilant. Speech is pressured and difficult to interrupt. He says, "This is all an unfair plot to get rid of me. I have contributed more to that company than anyone, and my business plan is pure genius." Blood pressure is 160/100, pulse is 108 and RR is 16. Pupils are dilated and reactive to light. Which of the following is the most likely explanation for this patient's behavior? a. Bipolar disorder b. Cocaine use disorder c. Delusional disorder, grandiose type d. Paranoid personality disorder e. Schizophrenia

B 78. Clues suggestive of cocaine abuse: • Sympathetic nervous system stimulation • Psychosis, personality changes, mood swings, panic attacks • Chest pain, headaches, rhinitis/epistaxis

1) A 15-year-old boy is brought to the office by his mother due to "odd behavior." Since moving to a new school 8 months ago, the boy has started talking to his imaginary friend Henry again. His mother reports that he had this imaginary friend for a few months in kindergarten. The boy has been spending more time alone, and he spends time laughing and talking to Henry. His teachers report that the patient has not been paying attention in class or completing his homework. He no longer spends time with his siblings and says he "would rather play video games with Henry." The patient states, "Henry is always with me and likes to comment on what I am doing." He has tried marijuana in the past but does not use other drugs. On mental status examination, the patient avoids eye contact and has a flat affect. Which of the following is the most likely explanation for the patient's behavior? a. Schizoid personality disorder b. Schizophrenia c. Schizophreniform disorder d. Schizotypal personality disorder e. Substance-induced psychotic disorder

B 8-month history of auditory hallucinations plus negative symptoms (social withdrawal, flat affect) resulting in significant functional decline = consistent with schizophrenia. Hallucinations = more common than delusions in pediatric schizophrenia.

83) A 28-year-old woman comes to the office for follow-up due to depression. She was prescribed fluoxetine after 4 weeks of depressed mood, loss of interest in activities, fatigue, hypersomnia,, and carbohydrate cravings. After 8 weeks of treatment, the patient feels partially better but continues to struggle with low energy and weight gain. Although she functions adequately at work, she spends the rest of the time at home, feeling too tired and unmotivated to go out with friends or exercise. The patient has a history of antidepressant treatment for recurrent depressive episodes that start in the late fall and improve by the spring, when she feels significantly better. She usually stops taking the medication for several months until the depression returns. Which of the following is the best next step in the management of this patient? a. Add aripiprazole b. Add bright light therapy c. Add valproate d. Switch to lithium e. Switch to phenelzine

B 83. Diagnosis = major depressive disorder with seasonal pattern (seasonal affective disorder); first line treatment = antidepressants and bright light therapy

87) A 58-year-old woman comes to the emergency department after developing a headache following a gourmet meal that included heavy sauces, smoked meats, and wine. On arrival, the patient is anxious and tremulous. Medical history includes recent major depression with psychotic features for which she takes a monoamine oxidase inhibitor as prescribed. Her depression has been in remission for the past 5 years. She does not use illicit drugs. Which of the following is the most likely finding on physical exam? a. Hyperreflexia b. Hypertension c. Hyperthermia d. Muscle rigidity e. Myoclonus

B 87. Diagnosis = adverse reaction between MAOI and tyramine-containing foods (wine, aged cheeses, cured meats) causing a sympathomimetic effect (HTN/hypertensive crisis) • Choices A & E: hyperreflexia and myoclonus are symptoms of serotonin syndrome, which can be caused when MAOIs are mixed with SSRIs • Choices C & D: hyperthermia and diffuse muscle rigidity are symptoms of neuroleptic malignant syndrome, which only occurs with antipsychotics

9) A 65-year-old woman comes to the office due to deteriorating memory. She used to pride herself on her sharp memory, but over the past 6 months, has been forgetting minor things. The patient has had to delegate household chores and cooking to her husband because of her poor memory and "cloudy thinking." "I have been feeling useless and worthless since retiring last year." The patient used to garden every day, but now spends her days watching television and barely has enough energy to eat. She's lost 5 kg (11 lb) over the last 2 months. On testing with the Montreal Cognitive Assessment, she scores 24/30 (normal: ≥ 26) with deficits in delayed recall and attention. Laboratory results and MRI of the head are unremarkable. Which of the following is the next best step in management? a. Donepezil b. Escitalopram c. Memantine d. Memory training and vitamin E e. Reassure that this is age-related memory change

B 9. Diagnosis = major depressive disorder (anhedonia, feelings of worthlessness, weight loss, low energy, impaired thinking/memory); severe depression in older adults can cause depression-related cognitive impairment; Tx = SSRI (&/or psychotherapy)

91) A 2-year-old boy is brought to the clinic by his mother, who is concerned about his language development. She says, "His speech isn't progressing and he does not interact with other children. His day care teacher says that he is shy and sits in a corner playing with his favorite truck." The family moved to a new home 3 months ago. In the exam room, the patient sits in a corner, avoids eye contact with the health care provider, and repeatedly spins the wheels of a toy truck. He does not respond to his name. He makes occasional grunting sounds but says no fully-formed words. When his mother tries to take his truck away, he begins screaming and pulls it back. Which of the following is the most likely diagnosis in this patient? a. Adjustment disorder b. Autism spectrum disorder c. Normal variation in development d. Selective mutism e. Social anxiety disorder

B 91. Features in this child consistent with ASD: speech delay, repetitive play, lack of social engagement

95) A 62-year-old woman comes to the office due to insomnia and depressed mood. The patient states that she feels angry and sad since her husband of 35 years unexpectedly announced that he wanted a divorce 2 months ago. She says "I was devastated because I thought we had a pretty good marriage. My whole life has been turned upside-down." The patient stays up late worrying about living alone and supporting herself financially. She recently started drinking wine before bedtime to help her fall asleep. Although she was regularly eating lunch with friends, the patient has canceled multiple times because she "can't bear to go through the story again and again." She has no suicidal ideation. The patient is tearful when discussing the divorce but brightens when asked about her grandchildren. Which of the following is the most likely diagnosis in this patient? a. Acute stress disorder b. Adjustment disorder c. Generalized anxiety disorder d. Major depressive disorder e. Persistent depressive disorder (dysthymia)

B 95. Onset of depressive symptoms following a major life stressor (divorce) and absence of severe enough symptoms to diagnose major depressive disorder = adjustment disorder (onset within 3 months of the stressor)

98) A 44-year-old woman requests a same-day appointment due to poor sleep. The patient says that when she goes to bed, her "brain can't shut off" and she feels "too wired" to fall asleep. She has been spending hours playing online games and buying things from the internet. The patient is well-known to the health care provider (HCP) and has a long history of bipolar disorder. Her medications include valproate and ziprasidone. Physical exam is normal. On mental status exam, the patient is very animated, laughs frequently, and speaks very rapidly. She has no hallucinations, and no delusions are elicited. The HCP increases the patient's valproate dosage and prescribes a 1-week supply of zolpdiem for sleep, with a planned follow-up in a week. At the close of the visit, the patient thanks the HCP profusely and presents a gift of a sterling silver pen, saying she is so appreciative that her visit could be scheduled on such short notice. Which of the following is the most appropriate response? a. I appreciate the gift and will continue to do my best to take care of you b. I appreciate your thanks and am happy to take care of you, but I am unable to accept the gift. c. Thank you, but I cannot accept gifts from patients unless they are inexpensive d. Thank you, but health care providers are prohibited from accepting any gifts e. Thank you for the gift, but please understand that it does not affect the care you receive

B 98. The HCP must consider that this patient's gift was purchased within the context of impaired judgement due to her hypomanic state, so it should be declined politely while still expressing appreciation

11) An 83-year-old woman is sent to the emergency department from her nursing home for evaluation of mental status changes. At baseline, she has mild memory impairment but is otherwise cognitively intact, calm, and cooperative. Over the past 24 hours, she has become increasingly combative and agitated and stayed up all night. Temperature is 99º F, BP is 110/80, pulse is 84, and RR is 18. Neurological exam is normal, but the patient is mildly disoriented. Without provocation, she strikes out at a nurses' aide standing next to her. Urinalysis is positive for nitrites and leukocyte esterase. Head CT scan is negative. In addition to starting antibiotics, which of the following medications is most appropriate to treat this patient's behavioral symptoms? a. Clozapine b. Doxepin c. Haloperidol d. Lithium e. Lorazepam

C 11. Acute treatment of agitation/psychosis associated with delirium = 1st-generation antipsychotics (or some 2nd-gen, e.g. quetiapine); appropriate for patients at risk of acute harm to self or others, after behavioral interventions have failed.

12) An 11-year-old girl is brought to the office due to disruptive behavior at home and at school. Her parents report that she gets in trouble for talking during class and not following instructions. Although the patient is of above-average intelligence, her grades are poor. She makes careless mistakes while rushing through tests and frequently forgets to hand in assignments. At home, she is easily distracted while trying to focus on her homework. The patient has frequent conflicts with her mother, who says, "Getting her ready for school in the morning is impossible. We're frequently late because she always misplaces her books." A medication with which of the following mechanisms of action is most appropriate for this patient? a. Antagonism of alpha-2 adrenergic receptors b. Antagonism of dopamine D2 receptors c. Increased availability of norepinephrine and dopamine d. Increased availability of serotonin and norepinephrine e. Positive allosteric modulation of GABA

C 12. Diagnosis = ADHD; stimulants = 1st-line treatment (MOA: block dopamine and norepinephrine reuptake at synapses in the prefrontal cortex and increase the release of norepinephrine and dopamine)

13) A 28-year-old woman comes to the office due to panic attacks. She was promoted at work 7 months ago and has experienced severe anxiety, trembling, sweating, chest tightness, and shortness of breath when giving presentations or meeting new clients. The patient says, "I have been anxious around people since I was a teenager, but it is getting worse. I know everyone can see that I'm extremely nervous in meetings and will think I'm incompetent. I'm afraid I'll make a fool of myself. The last time I had to speak, I became so anxious that I couldn't breathe and almost passed out." The patient is becoming increasingly concerned that she will lose her job, which is resulting in poor sleep. On mental status exam, she is visibly anxious throughout the interview, and avoids eye contact with the health care provider. Which of the following is the most likely diagnosis in this patient? a. Generalized anxiety disorder b. Panic disorder c. Social anxiety disorder d. Somatic symptom disorder e. Specific phobia

C 13. Social anxiety disorder: fear of negative evaluation, fear of embarrassment in social and performance situations, fear of scrutiny

15) A 46-year-old truck driver comes to the office for a follow-up visit for alcohol, amphetamine, and opioid addiction. The patient reports a reduction in substance use and cravings over the last 3 months with naltrexone therapy and participation in Alcoholics Anonymous meetings. He has been on a medical leave of absence from work to help him abstain from substance use and focus on recovery. The patient feels ready to return to work and brings a letter from his employer asking when he can return to work and if there are any restrictions or safety concerns. The patient has signed a release of information. Which of the following would be the most appropriate response to the employer? a. Patient confidentiality prohibits me from providing information on this case b. The patient is able to return to work, but random urine drug testing is recommended c. The patient is medically stable and may return to work d. The patient may return to work and should do well provided he takes his medication e. The patient's addiction is in remission and he is able to return to work.

C 15. HIPAA requires the minimum necessary release of information required to complete a request (disclosing ongoing treatment/issues is inappropriate)

16) A 37-year-old man is brought to the emergency department by police officers. He was found naked at a traffic signal, cursing at drivers and assaulting nearby pedestrians. The patient has a long history fo abusing multiple substances. He is agitated and alternates between screaming and pounding on the walls. Temperature is 100.9º F, BP is 148/100, pulse is 98, and RR is 16. Physical exam shows ataxia, nystagmus, and muscle rigidity. Which of the following is the next best step in the management of this patient? a. Dimercaprol b. Fomepizole c. Lorazepam d. Naloxone e. Varenicline

C 16. Diagnosis = PCP intoxication (agitation, delusions, psychosis, analgesia, aggression, nystagmus, HTN, tachycardia, disorientation); preferred treatment for PCP-related aggression (to maintain patient safety) = benzodiazepines

22) A 32-year-old man is brought to the emergency department after falling from a fourth floor balcony of his apartment building. A witness reports that the patient had stood on the balcony ledge for several minutes beforehand, looking down toward the street. The patient explains that he had a couple of beers and lost his balance while bird watching. He has no medical history, lives alone, and drinks 3-5 beers daily. Blood alcohol concentration is 0.005%. The patient is medically cleared. Mental status exam shows a thin, anxious man who is oriented to person, place, and time. The patient makes poor eye contact and gives brief responses, repeatedly insisting, "Everything's fine. I just want to go home." He says he is not suicidal. Which of the following is the next best step in the management of this patient? a. Arrange outpatient psychiatric follow-up for the next day b. Discuss options for antidepressant therapy and recommend outpatient psychotherapy c. Hospitalize the patient involuntarily d. Observe the patient in the emergency department for 24 hours and then reassess e. Perform a brief intervention for alcohol use

C 22. Despite patient's denial, he has likely attempted suicide and is at high risk for a repeated attempt. Outpatient management would be unsafe. Patient's risk factors: • Anxious affect (suggestive of depression) • Lives alone • Active substance use

25) A 20-year-old college student comes to the office due to persistent fatigue, irregular menstrual periods, and difficulty losing weight despite intensive exercise. Several times a week, the patient has episodes where she uncontrollably consumes large amounts of cookies and chips. She feels disgusted with herself afterward and subsequently does additional exercise. Although the patient hates her appearance and constantly compares herself to her slimmer friends, she denies feeling persistently depressed. BP is 100/60 and pulse is 92. Exam shows pharyngeal erythema and minimal parotid enlargement bilaterally. Potassium is 3.4 and amylase is 140. Pregnancy test is negative. Which of the following medications would be most effective in treating this patient? a. Buproprion b. Desipramine c. Fluoxetine d. Lisdexamfetamine e. No pharmacological treatment is effective

C 25. Diagnosis = bulimia nervosa; drug of choice = fluoxetine (best evidence of being most effective in combination with nutritional rehabilitation and psychotherapy

28) A 45-year-old man comes to the office for his annual physical exam. He moved to the area 3 weeks ago to work as a local television newscaster. The patient quit his last job while he was on-air because they hired a co-anchor and "there was no room for me." He became "extremely upset" last week when he saw an unflattering photograph of himself still featured on the station's website. The patient then crosses his arms dramatically and describes how he had previously quit another television job after his boss said he was difficult to work with because he was always taking over meetings. He bursts into tears and says, "I thought he was my friend, but friends don't stab you in the back." On mental status exam, speech is loud, but not pressured; facial expressions and hand gestures appear exaggerated. Which of the following is the most likely diagnosis in this patient? a. Antisocial personality disorder b. Borderline personality disorder c. Histrionic personality disorder d. Intermittent explosive disorder e. Narcissistic personality disorder

C 28. Histrionic personality disorder features: • Pattern of excessive emotionality and attention-seeking behavior, since early adulthood • Inappropriate, sexually seductive, or provocative behavior • Uses appearance to draw attention • Impressionistic, vague speech • Easily influenced; suggestible • Considers relationships more intimate than they actually are

3) A 59-year-old woman comes to the office due to upper respiratory symptoms for the past 3 days. She arrives an hour late for her appointment, blaming it on an emergency at work that "only I could take care of." The patient is angry that the provider refused to prescribe antibiotics for her over the phone, and she paces at the reception desk, insisting that the office staff work her into the schedule because "I'm very busy and must be seen immediately." The patient then talks at length about feeling depressed because her children no longer want to see her even though she is a "great mother." She is upset that her daughter recently canceled plans to visit in order to help a friend who was in a near-fatal motor vehicle collision. Which of the following is the best explanation for this patient's behavior? a. Borderline personality disorder b. Histrionic personality disorder c. Narcissistic personality disorder d. Paranoid personality disorder e. Persistent depressive disorder (dysthymia)

C 3. Narcissistic personality disorder (cluster B): grandiosity, need for admiration, sense of entitlement, lack of empathy; difficulty sustaining relationships

34) A 9-month-old boy is brought to the office by his mother for a routine visit. His mother is planning to return to work next month but is worried about dropping the patient off at day care because when someone tries to hold him, he cries and screams. He has become very "clingy"to his parents. The patient crawls and says "ma." During evaluation, he sits quietly on his mother's lap but cries when picked up by the nurse. When the nurse calls the patient's name, he turns his head and looks at his mother. Which of the following is the most likely explanation for this patient's behavior? a. Adjustment disorder b. Autism spectrum disorder c. Normal development d. Reactive attachment disorder e. Separation anxiety disorder

C 34. Stranger anxiety is normal; begins around 6 months, peaks at 8-9 months, and resolves by 24 months.

38) A 31-year-old woman comes to the office for follow-up of bipolar I disorder. The patient was diagnosed at age 20 and has had 2 hospitalizations for major depressive episodes. Herr mood has been stable on valproate for the past 2 years. The patient hopes to become pregnant and would like to stop her oral contraceptive soon. She does not want to risk another hospitalization and would prefer to keep taking medication during pregnancy. After a discussion of the options, the patient chooses to stop the valproate and switch to a different medication. Which of the following is the best treatment option for this patient? a. Buproprion b. Carbamazepine c. Lamotrigine d. Lithium e. Sertraline

C 38. Lamotrigine = favorable pregnancy safety profile; (buproprion/sertraline are also safe, but avoid antidepressant monotherapy in patients with bipolar)

41) A 35-year-old man comes to the office due to poor sleep and impaired concentration at work. He used to sleep well, but since returning from his last tour of military duty 6 months ago, he has had difficulty falling and staying asleep. The patient also has frequent nightmares and awakens in a state of panic. During the day, he feels tired and distracted. The patient says he sees images of explosions and hears his friend's voice calling to him for help. He often checks his doors in case anyone is breaking into the house. He does not feel depressed or have suicidal thoughts. Mental status exam shows a visibly tense and restless man whose eyes frequently scan the room. Which of the following is the most appropriate pharmacotherapy for this patient? a. Buspirone b. Lorazepam c. Paroxetine d. Quetiapine e. Trazodone

C 41. Diagnosis = PTSD; treatment = SSRI or SNRI, CBT

42) A 2-year-old boy is brought in for a routine well-child exam. The patient was born at term and has achieved all motor milestones normally. He started babbling at age 9 months and said his first words around 15 months. He then became relatively quiet and "obsessed" with stacking blocks and organizing his socks by color. He appears to be "in his own little world" and shows no interest in looking at people, including his family members. His parents rarely take him outside as it triggers severe tantrums and head-banging. Which of the following clinical features is typically seen in this patient's condition? a. Distress over ritualistic behaviors b. Excessive fears of embarrassment c. Inflexible adherence to routine d. Strong reading comprehension skills e. Vision impairment

C 42. Diagnosis = autism spectrum disorder • Choices A & B: characteristics of OCD • Choice D: cognitive skills vary among patients with ASD • Choice E: this patient's vision appears grossly normal (he is able to stack blocks/organize socks by color)

43) A 27-year-old woman comes to the office due to recurrent episodes of anxiety, palpitations, shortness of breath, tremors, and numbness and tingling in her hands. The patient describes breaking into a sweat suddenly and feeling dizzy several times a week. The last time it happened she was embarrassed as it occurred during a meeting at work. During the meeting, she ran out of the room because she felt short of breath, had a choking sensation, and her chest was pounding. The patient now feels anxious all the time because she fears having another episode. She does not feel depressed and reports no change in her sleep pattern or appetite. Physical exam, laboratory evaluation, and ECG are normal. Which of the following is the most appropriate pharmacotherapy for this patient? a. Buspirone b. Clonazepam c. Escitalopram d. Lorazepam e. Propanolol

C 43. Diagnosis = panic disorder (recurrent, unexpected anxiety attacks with physical symptoms); treatment = SSRI or SNRI

52) A 23-year-old man comes to the office at his girlfriend's insistence. She says that the patient's frequent snoring keeps her up at night and that in the last 4 months he has fallen asleep twice while they were talking. In this same period, the patient says that he has regularly fallen asleep in the afternoon while reading or watching television, but typically feels refreshed after a brief nap. The patient says that sometimes he hears a voice call his name prior to falling asleep. He also reports uncharacteristic episodes of clumsiness in which he has suddenly dropped objects or fallen to his knees. Which of the following is the most likely diagnosis in this patient? a. Delayed sleep-wake phase disorder b. Insufficient sleep c. Narcolepsy d. Obstructive sleep apnea e. REM sleep behavior disorder

C 52. Features of narcolepsy: • Recurrent lapses into sleep ≥ 3 times/week for ≥ 3 months • Cataplexy (loss of muscle tone precipitated by strong emotion), low levels of hypocretin-1 in CSF, and/or shortened REM sleep latency • Hypnagogic (upon falling asleep) or hypnopompic (upon awakening) hallucinations • Sleep paralysis

61) A 24-year-old woman is brought to the emergency department due to depression and recent suicidal thoughts. The patient has no suicidal ideation but feels depressed, exhausted, and unable to concentrate. She appears distracted and speaks slowly. For the past 3 nights, the patient attended parties where drugs and alcohol were present. She was energetic and talkative at the parties; however, last night she slept for 18 hours and had difficulty getting out of bed and missed work. Her boyfriend says she has had brief periods of acute depression over the past several months that lasted a few days and were charactized by low energy, hypersomnia, vivid unpleasant dreams, and increased appetite. Vital signs and physical exam are normal. Which of the following is the most likely diagnosis for this patient? a. Alcohol withdrawal b. Bipolar disorder c. Cocaine withdrawal d. Major depressive disorder e. Opiate withdrawal

C 61. Features of cocaine withdrawal: • Depression, fatigue, hypersomnia • Increased dreaming • Hyperphagia • Impaired concentration • Intense cravings • Chills, tremors, myalgias

65) A 19-year-old woman is brought to the emergency department after she was found trespassing at an animal shelter. The patient had keys to a rental van, as well as numerous collars and leashes. The patient reports that the shelter was going to perform dangerous experiments on the animals and that she is the only one who can save them. She quit her job a week ago to spend more time researching escape routes for the animals. The patient repeatedly interrupts the evaluation to yell at other patients and hospital staff walking by the room. She refers to herself as a "cat burglar" and then laughs loudly for a few minutes. Urine drug screen is negative. Which of the following additional features is most likely in this patient? a. Experiencing a sensation of bugs crawling on the skin b. Experiencing intrusive, unwanted thoughts c. Feeling well-rested despite minimal sleep d. Neglecting personal grooming e. Pausing for long periods prior to responding to questions

C 65. Diagnosis = manic episode (bipolar I); key feature of mania = decreased need for sleep • Choice A: tactile hallucinations are more associated with substance use disorders • Choice B: unwanted thoughts are a core feature of OCD • Choices D & E: these are negative symptoms, characteristic of schizophrenia

76) A 27-year-old woman comes to the office due to headaches and neck and back pain. She has been coping with pain symptoms for several years but notes that they have increased since she was recently let go from her job. The patient worries about finding another job but states that she is too tired or unmotivated to look for work on most days. She has been to the office twice in the past year for similar symptoms. At her last visit 4 months ago, laboratory testing, including a basic chemistry panel, thyroid function tests, and CBC, were normal. The patient says, "I doubt that anything will ever improve for me." Vital signs and physical exam are normal except for a 2.27 kg (5 lb) weight gain since her last visit. Which of the following is the best next step in management of this patient? a. Examine for point tenderness b. Explore the relationship of her symptoms to stress c. Inquire about suicidal thoughts d. Obtain a repeat TSH e. Offer reassurance and schedule regular follow-up

C 76. Patient's symptoms/presentation are consistent with somatic depression; workup of depression should include a screening for suicidal ideation

79) A 28-year-old woman is brought to the hospital due to depressed mood, disorganized behavior, and hearing voices. Over the past 2 months, the patient has developed worsening auditory hallucinations and now fears that her family has been replaced by imposters. She is afraid to leave the house, stays in bed most of the day, sleeps longer than usual, is not eating, and is unable to care for herself. The patient has had 4 hospitalizations for depressive and psychotic symptoms over the past 8 years. Between hospitalizations, she has had rare month-long periods during which she is not depressed but continues to hear voices. On mental status exam, the patient's mood is depressed, and she has current suicidal ideation. Which of the following is the most likely diagnosis in this patient? a. Bipolar I disorder with psychotic features b. Delusional disorder c. Schizoaffective disorder d. Schizophrenia e. Schizophreniform disorder

C 79. Features of schizoaffective disorder: • Major depressive or manic episode concurrent with symptoms of schizophrenia • History of delusions/hallucinations for ≥ 2 weeks in the absence of major of mood episodes • (Patient has 8-year history of psychotic symptoms with depressive symptoms, but also has month-long periods of just psychosis without depression)

8) A 42-year-old man is brought to the emergency department after he was found wandering aimlessly at an airport. The man is alert and answers questions appropriately; however, he appears to be confused about his identity and does not recognize the name on his Nevada driver's license in his wallet. The patient is perplexed and does not remember where he lives, how he got to the airport, his family members, or his profession. His cognitive function is otherwise intact. A search of his personal belongings reveals an airline ticket from Las Vegas to Arlington, Virginia. An emergency contact on the patient's phone leads to his wife in Nevada, who reports that her husband "just disappeared" after he found out that his father had passed away that morning. Which of the following is the most likely diagnosis? a. Acute stress disorder b. depersonalization/derealization disorder c. Dissociative amnesia d. dissociative identity disorder e. Transient global amnesia

C 8. Dissociative amnesia: • Inability to recall important personal info • Usually precipitated by traumatic/stressful event • Not explained by another disorder (e.g. PTSD, substance abuse)

80) A 28-year-old man is brought to the emergency department by his roommate, who is concerned about his change in behavior over the past 2 weeks. The roommate describes the patient as "a regular guy who is usually very responsible." Last week, the patient abruptly quit his job as a computer programmer and started placing large bets on an online gambling site because he was "sure to make millions." The roommate says that the patient has been staying up most nights scribbling notes for his autobiography on small scraps of paper. The patient says, "My new mission is to spread understanding." He denies any alcohol or drug use, which his roommate affirms. This patient is most likely to exhibit which of the following additional findings? a. Flat affect b. Poor hygiene c. Pressured speech d. Social withdrawal e. Thought blocking

C 80. Diagnosis = manic episode • Choices A, B, and E = symptoms of schizophrenia • Choice D = symptom of depression

89) A 54-year-old woman comes to the office due to anxiety, insomnia, and muscle tension. The patient reports feeling depressed and overwhelmed by multiple stressors in her life, including her father's death a year ago and a high-pressure job. For the past 2 months, she reports frequent tension headaches, neck pain, fatigue, and difficulty concentrating at work. At night she lies in bed worrying and has difficulty both falling and staying asleep. The patient has avoided socializing with her friends and says, "I just feel to down. Every time I am around them, I feel they must know I am a failure." She has lost 5 pounds and has lost interest in cooking and exercising. The patient has no prior psychiatric history. Physical exam and laboratory evaluation are unremarkable. The patient bursts into tears when speaking about her father. Which of the following is the most likely diagnosis? a. Adjustment disorder b. Generalized anxiety disorder c. Major depressive disorder d. Normal grief e. Social anxiety disorder

C 89. Anxiety can be an associated symptom of depression; this patient has enough symptoms of depression that she meets the criteria for major depressive disorder

92) A 27-year-old man comes to the office for follow-up after recent hospitalization due to a depressive episode with suicidal ideation. The episode remitted with intensive inpatient psychotherapy and the initiation of a new medication regimen a month ago. The patient has been home for a week and is feeling well. He has a history of migraine headaches and bipolar disorder. Vital signs are normal. Exam is significant for a maculopapular rash over the left scapula. Discharge paperwork from the previous week does not document a rash. Which of the following medications is the most likely cause of this patient's condition? a. Escitalopram b. Haloperidol c. Lamotrigine d. Olanzapine e. Quetiapine

C 92. Lamotrigine: anticonvulsant used as a mood stabilizer for bipolar depression; MC side effect = drug rash (rash is not a common side effect of antidepressants or antipsychotics)

45) A 23-year-old woman comes to the office for follow-up after a recent hospitalization. The patient had no known psychiatric history prior to being arrested by police 2 months ago after breaking into city hall to "find evidence of a secret organization plotting to take over the world." When police officers took her into custody, she was speaking so rapidly that they had difficulty understanding her. She was hospitalized and prescribed appropriate treatment; symptoms improved, and she was discharged 2 weeks later. Since then, the patient has been taking medications as prescribed and she has no symptoms. Vital signs are normal and physical exam is unremarkable. The patient's speech is regular and she appears calm. She is fully oriented and does not appear distracted. Laboratory studies are unremarkable except for serum calcium of 13.4. The patient is most likely taking which of the following medications? a. Aripiprazole b. Lamotrigine c. Lithium d. Quetiapine e. Valproate

C Diagnosis = bipolar manic episode (acute onset of rapid speech & symptoms of psychosis, followed by a return to normal function); treatment = lithium • Lithium side effects: hyperparathyroidism (causing hypercalcemia), nephrogenic diabetes insipidus, chronic kidney disease, hypothyroidism, dysrhythmias (in patients with risk factors for CAD)

10) A 4-year-old boy is brought to the office for a routine visit. He is growing well and talking in short sentences. He has started to play board games with his brothers and sister but occasionally hits his siblings when he loses a game. His parents ask if it is normal for him to occasionally fondle his genitals in public or try to touch his brother's genitals while bathing. The boy has been found playing with dolls in which they have kissed each other's private parts a few times. The parents are also concerned that the patient occasionally wets his bed at night. Which of the following behaviors is concerning in this patient? a. Becoming physically aggressive when losing a game b. Fondling one's own genitals in public c. Nocturnal enuresis d. Simulating oral sex using dolls e. Touching the genitals of a sibling while bathing

D 10. Abnormal sexual behavior in preadolescents: • Repeated insertion of objects into vagina/anus • Sex play involving genital-genital, oral-genital, or anal-genital contact • Use of force, threats, or bribes in sex play • Age-inappropriate sexual knowledge • (Choices B and E are normal sexual behaviors in toddlers)

100) A 23- year-old man is brought to the office due to increasingly odd behavior and progressive social withdrawal over the past year. The patient dropped out of college last year because of failing grades. Since returning to live with his parents, he stopped going out with his friends and sits in his room all day. For the past few months, he has been watching the same videos over and over, telling his mother that he is "listening for messages from secret channels." The patient says, "I'm fine. I don't know why everyone's so upset." His appetite, sleep, and energy level are normal. On exam, the patient appears anxious, avoids eye contact, and shows little emotion. His answers are very brief, and he asks if the interview is secretly being recorded. The patient has no suicidal ideation, symptoms of mania, or hallucinations. Which of the following is the most likely diagnosis in this patient? a. Delusional disorder b. Schizoaffective disorder c. Schizoid personality disorder d. Schizophrenia e. Schizotypal personality disorder

D 100. Patient's symptoms, lasting ≥ 6 months = consistent with schizophrenia • Patient's negative symptoms: apathy, abolition, flat affect, alogia (brief answers), asociality

14) A 32-year-old man comes to the office due to severely depressed mood. For the pas month, the patient has had no energy. He has missed several days at work because he cannot motivate himself to get out of bed. The patient is sleeping 12 hours a day and his appetite is poor. He has no delusions or hallucinations. The patient has a history of depression in his 20s that responded poorly to fluoxetine. He was hospitalized briefly 2 years ago after being arrested for aggressive behavior when police attempted to restrain him when he entered a government building without authorization; he wanted to explain his "strategy for world peace" and was speaking rapidly and often illogically. Following discharge, he did not follow up with treatment. Which of the following is the most appropriate pharmacotherapy for this patient? a. Bupropion b. Nortriptyline c. Paroxetine d. Quetiapine e. Venlafaxine

D 14. Patient's current symptoms = major depressive episode; history of hospitalization for impulsive/aggressive behavior, rapid speech, and grandiose delusions = suggestive of prior manic episode; diagnosis = bipolar I disorder; quetiapine (2nd-generation antipsychotic) = effective treatment for bipolar I • Other bipolar I treatments: lurasidone (2nd-gen antipsychotic), lithium (mood stabilizer), lamotrigine, valproate

29) A 62-year-old woman comes into the office accompanied by her daughter, who is concerned that her mother is depressed. The patient has become more withdrawn and socially isolated after her husband's unexpected death from a heart attach 2 years ago. She still misses her husband terribly and thinks about him constantly. The patient says, "I can't believe he is gone." She feels guilty that she did not recognize his heart condition and blames herself for not insisting that he get medical care earlier. The patient does not have sleep or appetite disturbance but sleeps on the couch because she cannot bear to lie in the bed that she and her husband shared. She has topped playing golf and attending concerts - activities that they had enjoyed together. The patient has no suicidal ideation. She continues to work at her part-time job and help her daughter with the grandchildren. Which of the following is the most likely diagnosis in this patient? a. Dependent personality disorder b. Major depressive disorder c. Normal grief d. Persistent complex bereavement disorder e. Post-traumatic stress disorder

D 29. Persistent complex bereavement disorder features: • Grief lasting >6 to 12 months • Symptoms do not increase in intensity over time • Persistent yearning for the deceased • Difficulty accepting the death • Preoccupation with circumstances of the death • Significant social and occupational impairment • Thoughts of dying involving a wish to join the deceased (active suicidality is common)

33) A 73-year-old man with bipolar disorder is brought to the emergency department by his daughter, who has been visiting him for several days. She reports that "he's not himself." The patient has become increasingly confused, has bilateral hand tremors, has difficulty walking straight,and has vomited over the past few days. Two weeks ago,he saw his new primary care provider, who added hydrochlorothiazide to the patient's medication regimen. The patient has been psychiatrically stable for many years and has been seeing a psychiatrist, who has prescribed mood stabilizers and antipsychotics. He is disoriented, ataxic, and has a generalized seizure that lasts 2-3 minutes. Which of the following medications is the most likely cause of this patient's symptoms? a. Buproprion b. Haloperidol c. Lamotrigine d. Lithium e. Valproic acid

D 33. Thiazide diuretic can decrease the renal clearance of lithium, causing toxicity: • GI symptoms • Confusion/disorientation • Ataxia/tremor • Seizures

37) A 72-year-old man comes to the office for a health maintenance visit. He retired last year to care for his wife of 50 years, who has Alzheimer's disease. He gets up frequently during the night to assist her, fearing that she will wander and fall. She requires his assistance with dressing, feeding, and self-care. The patient has stopped socializing with friends and explains, "This is my new full-time job. I don't mind answering her same questions multiple times, but I lose my patience when she starts yelling at me for no reason or accusing me of being unfaithful I know she cant help it, but it is hard to take. I don't know what else I can do." In addition to providing support, which of the following is the most appropriate response to the patient? a. Given your difficulty in caring for her, I think it's time to consider nursing home placement b. I recommend supportive psychotherapy to help you cope with your frustration c. It's important to understand that her behavior is a symptom of dementia and not directed at you d. Let's discuss options for services that can help you care for your wife e. Low-dose antipsychotics may decrease your wife's agitation and improve her sleep

D 37. Diagnosis = caregiver distress (sleep disturbance, anxiety, social isolation, increasing frustration); reduce burden/prevent burnout by connecting caregivers with services (respite care, support groups, etc.)

39) A 37-year-old man is brought to the emergency department after his sister found him unresponsive. Several days ago, he was discharged from an inpatient facility where he was treated for multiple substance use disorders and long-standing bipolar disorder. The patient's sister states that he has been depressed and has not taken his prescribed medications since discharge. Temperature is 98.1º F, BP is 80/40, pulse is 62, and RR is 10. He is obtunded and unresponsive to verbal commands. The pupils are 1 mm and respond sluggishly to light. Fingerstick blood glucose is 72. The patient is ventilated, and supplemental oxygen and IV fluids are initiated. Which of the following is the best next step in the management of this patient? a. Buprenorphine b. Flumazenil c. Intravenous glucose d. Naloxone e. Naltrexone

D 39. Diagnosis = acute opioid intoxication/overdose (decreased level of consciousness, reduced respiratory rate, miosis); patients recently discharged from inpatient treatment are at higher risk of overdose due to loss of tolerance of opioids; treatment = naloxone (faster onset than naltrexone)

64) A 37-year-old woman comes to the office due to low energy and fatigue. She recently received a promotion at work and was very excited and motivated at first, but now she finds herself procrastinating and having difficulty concentrating. For many years, the patient has had unpredictable "mood swings" consisting of periods of low energy and "up periods" in which she feels more energetic and optimistic. These vary in length from days to weeks and have no clear relationship with situations in her life. She has no history of major depressive episodes or psychosis. She drinks 1-2 glasses of wine several times a week. Mental status exam shows a sad affect and some lapses in concentration, although the patient brightens easily. Which of the following is the most likely diagnosis in this patient? a. Bipolar I disorder b. Bipolar II disorder c. Borderline personality disorder d. Cyclothymic disorder e. Substance-induced mood disorder

D 64. Features of cyclothymic disorder: ≥ 2 years of fluctuating, mild hypomanic and depressive symptoms that do not meet criteria for hypomanic or major depressive episodes • Choice B: bipolar II = ≥ 1 hypomanic episode with ≥ 1 major depressive episode

4) A 42-year-old man comes to the office for a follow-up. Last year, the patient was hospitalized for acute gastritis. During the hospitalization, he went into alcohol withdrawal that was treated with chlordiazepoxide. The patient was abstinent for 2 weeks following discharge, but then started drinking again. Over the past several months, he has been drinking 6-10 beers daily. The patient says "I want to cut down, but the cravings are too strong." He fears he will lose his job and family if he continues to drink. Vital signs and physical exam are normal. Which of the following is the most appropriate pharmacotherapy for this patient's alcohol use disorder? a. Buprenorphine b. Bupropion c. Chlordiazepoxide d. Naltrexone e. Varenicline

D 4. Pharmacotherapy for alcohol use disorder: • Naltrexone: decreases cravings, reduces heavy drinking days, increases days of abstinence, can be initiated while patient is still drinking o CI in patients taking opioids (can precipitate withdrawal) o CI in patients with acute hepatitis or liver failure • Acamprosate (preferred for patients with liver disease or opioid use, CI in patients with renal impairment) • Disulfiram (only used second-line; patient must be highly-motivated)

46) A 26-year-old man is brought to the office by his former girlfriend, who became alarmed after receiving a message in which he threatened suicide. The patient says he went into an overwhelming state of panic and depression when he heard that his former girlfriend was dating a new man and "wanted to get back at her." The patient has a history of "mood swings" since his teens and rage attacks when he feels that people are unsupportive. He has no suicidal intent or plan. He describes his mood as a "deep pit of emptiness." Physical exam is normal except for cigarette burns on his thighs that he explains as self-harming behavior performed to relieve tension. Which of the following is the best treatment for this patient? a. Citalopram b. Lithium c. Olanzapine d. Psychotherapy e. Venlafaxine

D 46. Diagnosis = borderline personality disorder (unstable relationships, mood lability, impulsivity, self-injury; often present to healthcare during interpersonal crises); treatment = psychotherapy (dialectical behavioral therapy) - meds not effective

47) A 32-year-old man comes to the emergency department saying, "I feel awful; I need help." The patient says he was feeling fine until last night when he developed abdominal cramps that were initially mild but gradually worsened in intensity. Since this morning, he has had 4 loose stools. The patient has also had persistent nausea but no vomiting. Medications include paroxetine and clonazepam. Temperature is 97.9º F, BP is 144/98, pulse is 88 and RR is 16. Physical exam shows a diaphoretic, thin man in considerable distress. The pupils are dilated. This patient's clinical presentation is most consistent with withdrawal from which of the following? a. Alcohol b. Clonazepam c. Cocaine d. Heroin e. Paroxetine

D 47. Opioid (e.g. heroin) withdrawal: • Symptoms begin within 6-12h, peak in 36-72h • Myalgia, arthralgia, n/v, diarrhea, abdominal cramps, rhinorrhea, lacrimation, sweating, mydriasis, hyperactive bowel sounds, tachycardia, irritability, yawning

56) A 25-year-old woman is brought to the office by her husband due to intense mood swings and unusual behavior. She has rapid mood shifts that last a few hours. Although the patient is usually quiet and reserved, her husband says that she can suddenly become enraged without provocation. She does not remember these episodes. Her husband once found her at a local bar dressed in a seductive outfit; when he called her name, she did not respond and walked away. At home, the patient denies having been at the bar and hid behind a chair, crying. When she was finally convinced to come out, she spoke in a childlike voice, and said "leave me alone." The patient was sexually abused from age 5-16. She has difficulty giving a detailed history and has many gaps in her memory. The patient's mood is anxious, and she hears voices that she describes as coming from inside her head. Which of the following is the most likely diagnosis? a. Bipolar disorder b. Depersonalization/derealization disorder c. Dissociative amnesia d. Dissociative identity disorder e. Schizoaffective disorder

D 56. Features of dissociative identity disorder: ≥ 2 distinct identities, memory loss, history of childhood trauma

62) A 74-year-old woman is brought to the office by her daugter due to increasing forgetfulness and behavioral changes. Her daughter has noticed increasing forgetfulness over the past 3 years. Over the past year, the patient has stopped doing the daily crossword, has had difficulty remembering to eat and clean herself, and has occasionally failed to recognize her grandchildren. Over the past month, the patient has become more withdrawn. She no longer watches her favorite television shows and seems less interested in her family. Montreal Cognitive Assessment is 22/30 (normal: ≥ 26). Serum TSH, CBC, and vitamin B12 levels are normal. MRI of the brain shows age-appropriate cerebral atrophy. Which of the following is the best next step in the management of this patient? a. Duloxetine b. Fluoxetine c. Reassurance and follow-up only d. Rivastigmine e. Selegiline

D 62. Diagnosis = Alzheimer's disease; 1st-line treatment = cholinesterase inhibitor (e.g. donepezil, galantamine, rivastigmine)

63) A 27-year-old woman comes to the office due to intermittent abdominal cramping for the past few weeks. The patient has had no nausea, vomiting, or changes in bowel movements. Her stress level has been higher than normal since her boyfriend broke off their relationship a month ago. The patient says, "I did everything to please him and make him happy, and he abandoned me. I don't know how to go on without him." Her boyfriend managed her finances and recently helped her apply for a new job. The patient is unsure about switching jobs now and canceled her interviews to "avoid more confusion." She recently moved in with a friend and hopes this person can help her "sort everything out." Physical exam is normal. Which of the following is the most likely explanation for the patient's behavior? a. Acute stress disorder b. Adjustment disorder with anxiety c. Borderline personality disorder d. Dependent personality disorder e. Somatic symptom disorder

D 63. Features of dependent personality disorder (cluster C): submissive, clingy, indecisive, fearful of being on one's own, needs to be taken care of

67) A 27-year-old man comes to the office due to new-onset depression symptoms. The patient recently earned his master's degree but has struggled to find employment. Over the last 3 months, he has developed worsening dysphoria and often struggles to get out of bed in the morning. He sleeps 14-16 hours each night, feels exhausted throughout the day, has very little appetite, and feels profoundly hopeless and worthless. A decision is made to start fluoxetine for his major depressive disorder. Which of the following early adverse effects should be discussed with this patient prior to prescribing the medication? a. Dizziness, involuntary muscle jerks, and weight gain b. Drowsiness, ataxia, and behavioral disturbances c. Dyslipidemia, hyperglycemia, and sedation d. Headache, insomnia, and anxiety e. Urinary retention, dry mouth, and blurred vision

D 67. Choice A: side effects of MAOIs; choice B: side effects of benzodiazepines; choice C: side effects of 2nd-gen antipsychotics; choice E: side effects of TCAs

7) A 34-year-old man comes to the clinic for follow-up of schizophrenia. The patient has had multiple trials of antipsychotic medications with minimal improvement and is now taking aripiprazole. Despite adherence to his medication regimen, he has persistent psychotic symptoms. The patient has been unable to work, hears voices throughout the day, and is too paranoid to attend group therapy. He has no other medical conditions. After a discussion of the risks and benefits, the patient agrees to a trial of clozapine. Which of the following should be performed with use of this medication? a. Baseline and periodic liver function tests b. Baseline electrocardiography and serum electrolytes c. Periodic clozapine plasma levels d. Regular CBCs e. Regular thyroid and kidney function tests

D 7. Clozapine adverse effects: agranulocytosis/neutropenia; CBC monitoring is mandatory for routine absolute neutrophil counts (clozapine is 2nd-line for this reason)

81) A 32-year-old woman comes to the office due to increased urinary frequency and burning. Medical history is unremarkable except for 2 previous UTIs and several old injuries, including a broken arm and dislocated shoulder. She lives with her boyfriend of 3 years. During the exam, the health care provider notices bruises on the patient's arms, abdomen, and breasts. At first, she explains that she slipped and fell in the shower. When asked how things are at home, the patient mentions that her boyfriend has a bad temper and that they argue frequently. She reluctantly discloses that he has hit her several times when intoxicated; the last time was a week ago. In addition to acknowledging the abuse and providing support, which of the following is the most appropriate next step in the management of this patient? a. Ask why the patient remains in the relationship b. Encourage the patient to leave her boyfriend c. Encourage the patient to press charges d. Identify a place that the patient can go in an emergency e. Refer the patient for counseling

D 81. Patients who disclose intimate partner violence and who are not in immediate danger should be encouraged to make a safely plan (place to go, emergency kit with documents and essential items)

93) A 61-year-old man is brought to the office by his daughter for evaluation of memory impairment that she first noticed 3 months ago. The daughter reports that the patient cannot remember the content of conversations almost immediately after they have finished, but his memories of events from years ago are intact. The patient is asked how he got to the office. He replies, "I got in the car, drove for 2 hours, and then hopped on a trolley." The daughter says, "He lives 10 minutes away from here and I picked him up in my car." The patient appears indifferent to his daughter's concerns and does not believe there is anything wrong. Medical history includes hypertension and multiple hospitalizations for delirium tremens. Blood pressure is 137/85 and pulse is 78. Given a memory test, the patient recalls 0 of 3 items after 5 minutes but confidently names 3 inaccurate items. Which of the following is the most likely cause of this patient's condition? a. Alzheimer's disease b. Depression-related cognitive impairment c. Frontotemporal dementia d. Korsakoff syndrome e. Vascular dementia

D 93. Korsakoff syndrome = extreme B1 deficiency resulting from EtOH/chronic malnutrition; characteristic findings: • Retrograde and anterograde amnesia with preserved remote long-term memory • Confabulation to fill in memory gaps • Lack of insight • Commonly seen in patients with history of Wernicke encephalopathy

94) A 52-year-old woman comes to the office for a routine exam. During the exam, the patient confides to the health care provider that she has been "down" since her youngest child left for college 2 months ago. The patient is worried about her daughter being away from home for the first time. At work, the patient occasionally has lapses of concentration when worrying about her daughter, but it has not affected her productivity. She still enjoys going out with her husband but says all they do is talk about their kids. Which of the following is the most likely explanation for this patient's condition? a. Adjustment disorder with depressed mood b. Generalized anxiety disorder c. Major depressive disorder d. Normal sadness e. Persistent depressive disorder (dysthymia)

D 94. This patient does not meet the full criteria (SIGECAPS) for any disorder

2) A 42-year-old man is brought to the emergency department by his wife after he assaulted her. The patient has not slept or eaten for days; he became agitated and started accusing her of plotting to murder him. The patient also feels as though bugs are crawling under his skin. He had a prior psychiatric hospitalization 8 months ago when he was admitted for paranoid delusions and visual hallucinations. He is uncooperative, speaks rapidly and loudly, gets up to pace during the interview, and shouts "I don't trust any of you; you're in this together." Temperature is 100º F, BP is 140/90, pulse is 104, and RR is 20. Exam shows a thin, diaphoretic man with poor grooming and dentition. He clenches his teeth, picks at his skin, and has multiple sores on his face and body. Which of the following is the most likely diagnosis in this patient? a. Alcohol withdrawal b. Bipolar I disorder c. Delusional disorder, somatic subtype d. Methamphetamine use disorder e. Schizophrenia

D Methamphetamine intoxication: • Euphoric/dysphoric mood (anxiety, irritability) • Agitation (restlessness, tremor) • Bruxism/poor dentition • Psychotic symptoms (delusions, hallucinations, "bugs crawling under the skin", excoriations from skin picking) • Anorexia • Decreased need for sleep • Sympathomimetic effects (tachycardia, hypertension, mydriasis, diaphoresis)

26) A 72-year-old man is brought to the office due to visual disturbances that are causing significant distress. The patient describes seeing prowlers in the bushes outside of his windows at night that disappear when he moves in for a closer look. His daughter could not see these people. The patient has Parkinson's disease, for which he takes carbidopa-levodopa, and generalized anxiety disorder, for which he takes sertraline. Vital signs are normal. The patient is alert and oriented. He has a mild resting tremor and minimal rigidity. Movements are slightly slowed. Which of the following is the best next step in the management of this patient's visual disturbances? a. Add benztropine b. Add pramipexole c. Increase carbidopa-levodopa d. Increase sertraline e. Reduce carbidopa-levodopa

E 26. Psychosis in a patient with Parkinson's can be related to the disease, medication, or both; 1st step in management of PD psychosis = review medications and consider a cautious dose reduction of carbidopa-levodopa (add low-potency 2nd-gen antipsychotic [e.g. quetiapine] if patient cannot tolerate carbidopa-levodopa dose reduction)

31) A pharmaceutical company sales representative comes into the office to provide information on a new discount program for the company's antidepressant medication. One of the office staff members performs a search of the practice's electronic record system; she finds 2 patients on the schedule for the following week who are taking the drug and provides the names and addresses to the sales representative so that he can send information about the program. Later that day, when the clinician is notified of what happened, she immediately instructs the sales representative not to contact the patients and to delete their information from his computer. Which of the following is the next best step in addressing this situation? a. Contact the patients within 60 days to determine if any marketing materials were sent b. No additional action is necessary c. Notify the patients verbally of the breach of confidentiality at their next visit d. Notify the pharmaceutical company to initiate disciplinary action against the representative e. Send a written letter to the patients explaining what information was released and who received it

E 31. Breach of privacy = disclosure of PHI to an unauthorized person/for an improper purpose; patient must be notified in writing (describe the breach, what PHI was disclosed, and what actions are being taken to mitigate)

32) A 15-year-old boy is brought to the emergency department after he was found unconscious on his bedroom floor. He was seen walking upstairs 5 minutes before he was found. His father was able to rouse him quickly, but the patient continued to feel drowsy and experienced dizziness and headache. At the time of evaluation an hour later, the patient's symptoms have resolved, and he does not know why he lost consciousness. He has no chronic medical conditions. Vital signs and mental status exam are normal. There is a mild rash around the mouth. ECG is normal. Which of the following substances is the most likely cause of this patient's presentation? a. Alcohol b. Benzodiazepines c. Cocaine d. Heroin e. Inhalants

E 32. Dermatitis is seen in inhalant abuse due to chemical exposure around the mouth/nostrils

36) A 63-year-old man comes into the office for a follow-up examination of hypertension. At his last appointment, he revealed that he had increased his intake of 12-oz cans of beer from 3 to 6 cans a day due to stress at work. The patient was advised on his last visit to seek help for his alcohol use because it could be causing his elevated BP and has many other negative health risks. Today, the patient states, "I thought about what you said. I know my alcohol use has gotten out of hand and is affecting my health. My wife and daughter also say that I need to stop drinking. I have decided to look into treatment options and get some help." Which of the following best describes this patient's stage of behavioral change? a. Action b. Contemplation c. Maintenance d. Precontemplation e. Preparation

E 36. 5 stages of change: • Precontemplation (denial, no intention to change) • Contemplation (aware of the problem, not yet committed to a plan) • Preparation (decision and preparation to take action to change) • Action (active change/behavioral modification) • Maintenance (behavioral change over the long term)

5) A 17-year-old girl comes to the office due to "disturbing images" she has seen over the past year. She says, "I'd be standing in the kitchen helping my mother chop vegetables, and terrible images of stabbing her in the back with a knife would pop into my head." The patient is overwhelmed with anxiety each time this occurs, resulting in putting the knife down and running out of the kitchen. She counts down from 5 to 1 several times to try to "get rid" of the thoughts. The patient has a tense and anxious affect, and becomes tearful when discussing how she would never want to hurt her mother. She reports no depressed mood or suicidal ideation. Which of the following is the most appropriate pharmacotherapy for this patient? a. Aripiprazole b. Buspirone c. Haloperidol d. Lorazepam e. Sertraline

E 5. Diagnosis = OCD; treatment = SSRI, CBT

50) A 19-year-old man is brought to the office by his mother due to sad mood and declining grades at college. The patient feels depressed and hopeless about the future. Based on a complete history, major depressive disorder is diagnosed, and treatment is discussed. The patient is willing to take medication, but his mother is concerned that antidepressants can cause people to become suicidal. Which of the following is the most appropriate response? a. Some classes of antidepressants have a lower risk of increasing suicidal thoughts, and those are prescribed first to adolescents b. There is a slightly increased risk of suicidal thoughts in adolescents, and antidepressants are prescribed only when psychotherapy fails c. There is a slightly increased risk of suicidal thoughts in young people treated with antidepressants but only in those age < 18 d. There is no association between antidepressants and increased risk of suicidal thoughts in adolescents e. The slightly increased risk of suicidal thoughts with antidepressants in this age group should be weighed against the risk of completed suicide in untreated depression

E 50. 2007 FDA black box warning: all patients < 25y should be informed about the small risk of becoming suicidal during initial antidepressant treatment (increased risk of suicidal thoughts and behaviors, not completed suicide); patients should be closely monitored for suicidality at the beginning of antidepressant pharmacotherapy

53) A 6-year-old boy is brought to the office due to behavioral difficulties at school. He has been hitting other children and breaking their toys. The patient was adopted by his grandmother a year ago after his parents died in a motor vehicle collision. He attends the same school but no longer wants to play with his friends or join group activities. The patient wakes up a few week crying about monsters and asking to sleep in his grandmother's room. He is meeting appropriate developmental milestones. The patient is restless and rams a toy truck into a stuffed animal repeatedly while his grandmother speaks. He makes poor eye contact and answers most questions by saying "I don't know." Which of the following is the most likely diagnosis? a. Age-appropriate behavior b. Attention deficit hyperactivity disorder c. Autism spectrum disorder d. Oppositional defiant disorder e. Post-traumatic stress disorder

E 53. Features of PTSD in children (≥ 1 month): • Direct/indirect exposure to traumatic event • Nonspecific nightmares • Intrusive thoughts/repetitive themes in play • Emotional dysregulation (anger, irritability, aggression) • Emotional detachment/social withdrawal • Sleep disturbance • Anhedonia • Separation anxiety/fear of strangers

57) A 34-year-old woman comes to the office due to anxiety. She describes anxiety since childhood and now has frequent anxiety attacks characterized by sudden onset of intense fear, sweating, chest pressure, and difficulty breathing. The patient first experienced these attacks when she was bitten by a dog at age 12. Her fear of dogs subsided, but during college, she continued to have attacks and was treated with fluoxetine for 4 years, with good response. Over the past year, the patient has had anxiety attacks 2 or 3 times a week. Last week, she had an attack while at work and had to rush out of a meeting because she thought she was going to faint. Occasionally, the patient has attacks while relaxing at home. A diagnosis of panic disorder is considered. Which of the following features of this patient's history is most helpful in making this diagnosis? a. Development of agoraphobia b. Onset following a significant trauma c. Positive response to a selective serotonin reuptake inhibitor d. Prominent somatic symptoms e. Unexpected nature of some of the anxiety attacks

E 57. Key feature suggestive of panic disorder = some attacks are untriggered/unexpected • Choice A: not all patients with panic disorder develop agoraphobia • Choice B: onset of panic attacks following a traumatic event is more suggestive of specific phobia • Choice C: SSRIs are widely effective in a range of mental disorders • Choice D: somatic symptoms can be seen in other anxiety disorders

60) A 51-year-old attorney comes to the office due to sleep difficulties. She is a partner in a law firm and finds her job stressful but enjoyable. The patient goes to bed at 12:00 AM and has trouble falling asleep. When she cannot sleep, she watches the clock, tosses and turns, and worries about being able to function the next day, eventually falling asleep around 3:00 AM before having to wake up at 7:00 AM for work. She reports feeling "exhausted" by 6:00 PM and takes a 20-minute "power nap." The patient's job requires long hours, and she usually eats dinner late. She also has been more irritable lately and feels down at times but attributes this to "sleep deprivation." Which of the following is the most likely cause of the patient's symptoms? a. Delayed sleep phase syndrome b. Generalized anxiety disorder c. Perimenopausal sleep changes d. Persistent depressive disorder (dysthymia) e. Poor sleep hygiene

E 60. Patient's poor sleep hygiene habits: late afternoon naps, working late hours, eating late dinner, staying in bed when she cannot fall asleep

66) A 28-year-old graduate student is brought to the emergency department by 3 police officers after causing a disturbance in the dean's office. The student has not slept in a week because he is developing a cure for cancer that will result in a Nobel Prize. The patient has "endless energy" and "a million ideas." He has difficulty focusing on answering questions at times and becomes irritable and hostile when efforts are made to redirect him. The patient was diagnosed with depression at age 19. His father has adult polycystic kidney disease. Blood pressure is 142/82. Physical exam is unremarkable. Serum creatinine is 1.8. Which of the following medications is most appropriate for long-term management of this patient's illness? a. Buproprion b. Haloperidol c. Lorazepam d. Topiramate e. Valproate

E 66. Diagnosis = bipolar I; 1st-line maintenance treatment = lithium or valproate

70) A 59-year-old woman is brought to the emergency department by her son, who found her confused, shaking, sweating profusely, and unsteady on her feet. The patient has seasonal allergies, depression, and anxiety. Two weeks ago, fluoxetine was discontinued due to minimal improvement in her symptoms of depression; she was started on phenelzine a few days ago. Her other medications include diphenhydramine and lorazepam. Temperature is 101º F, BP is 160/90, pulse is 116, and RR is 24. The patient is oriented to person and place, but not time. She is agitated, diaphoretic, and tremulous. Mucous membranes are dry. Abdominal exam shows increased bowel sounds. Deep tendon reflexes are increased. There is some muscular rigidity in the lower extremities. Which of the following is the most likely diagnosis? a. Anticholinergic toxicity b. Benzodiazepine withdrawal c. Fluoxetine withdrawal d. Neuroleptic malignant syndrome e. Serotonin syndrome

E 70. Diagnosis = serotonin syndrome caused by interaction between MAOI (phenelzine) and SSRI (fluoxetine); should wait 14 days after stopping MAOI before starting SSRI • Serotonin syndrome triad: AMS (delirium, confusion, anxiety, restlessness), autonomic dysregulation (diaphoresis, tachycardia, HTN, hyperthermia, bowel hyperactivity, mydriasis), neuromuscular hyperactivity (hyperreflexia, tremor, rigidity, myoclonus)

71) An 18-year-old man is brought to the clinic due to increasingly bizarre behavior over the past 2 months. The patient is in his first semester at college and has had difficulty adjusting to being away from home. For the past few weeks, he has not been attending classes, and he no longer socializes with friends. The patient spends most of his time alone and eats very little because he believes the cafeteria food is poisoned. He has no psychiatric history. He appears disheveled with unwashed hair, makes poor eye contact, and shouts "leave me alone" in response to unseen people. Which of the following is the most likely diagnosis in this patient? a. Adjustment disorder b. Brief psychotic disorder c. Major depressive disorder with psychotic features d. Schizophrenia e. Schizophreniform disorder

E 71. Features of schizophreniform disorder: same symptoms as schizophrenia but lasting ≥ 1 month and < 6 months

73) A 6-year-old girl is brought to the office by her parents for evaluation of odd behaviors. For the past year, the patient has had brief episodes of repetitive blinking. The blinking worsens when she is excited and subsides when she is watching television. Due to the episodes, she has difficulty paying attention in class and plays alone because other children make fun of her. During evaluation, the patient intermittently squeaks and scrunches her nose; when asked about this, she says she cannot stop pretending to be a bunny. Her parents state that she has been "acting like a rabbit" every day for the past month. Physical exam, including ocular exam, is unremarkable. Which of the following is the most likely diagnosis in this patient? a. Absence seizures b. Attention deficit hyperactivity disorder c. Autism spectrum disorder d. Persistent motor tic disorder e. Tourette syndrome

E 73. Diagnostic criteria for Tourette's: multiple motor tics (e.g. blinking, scrunching nose) and ≥ 1 vocal tic (squeaking)

74) A 32-year-old woman with bipolar disorder comes to the office for a follow-up. The patient was diagnosed and treated for a manic episode at age 29 and has been stable on a combination of lithium and risperidone. Her mood has been "fine" overall, but she has been "stressed" about work. After a staff meeting a few weeks ago, her supervisor said she appeared angry because she was repeatedly frowning while others were presenting. Physical exam is significant for occasional grimacing and slow inversion and tapping movements of her right foot. Lithium level is 0.9 (normal: 0.6-1.2). Which of the following is the best next step in management of this patient? a. Add benztropine as needed b. Add daily propanolol c. Discontinue lithium and start valproate d. Maintain current regimen e. Taper and discontinue risperidone

E 74. Diagnosis = tardive dyskinesia (TD) caused by risperidone • TD is most common with 1st-gen antipsychotics and with risperidone • Features of TD = involuntary movements: orofacial dyskinesia (tongue protrusion, lip smacking, grimacing), limb dyskinesia (dystonic postures, foot tapping, chorea), trunk dyskinesia (rocking, thrusting, shoulder shrugging) • Treatment for TD: discontinue the offending med and switch to a 2nd-generation antipsychotic with lower risk of TD

82) A 31-year-old woman comes to the office for follow-up treatment of major depressive disorder. Five months ago, the patient attempted suicide after learning that her husband had filed for divorce. She was admitted to a psychiatric unit, where she was treated with fluoxetine 20 mg and discharged 2 months later. A month following discharge, the fluoxetine dosage was increased. The patient now says that her suicide attempt was "stupid" and denies suicidal ideation. However, despite taking maximum doses of fluoxetine for 8 weeks, she still feels very depressed and reports minimal improvement in her symptoms. She also has a history of bulimia nervosa. Mental status exam reveals poor eye contact, constricted affect, and soft speech. Which of the following is the most appropriate management of this patient's depression? a. Continue fluoxetine for an additional 4 weeks and reevaluate b. Discontinue fluoxetine and refer for electroconvulsive therapy c. Switch from fluoxetine to buproprion d. Switch from fluoxetine to nortriptyline e. Switch from fluoxetine to venlafaxine

E 82. Patients who fail ≥ 6 weeks of SSRI should switch to a different class of antidepressant (e.g. SNRI - venlafaxine) • Choice B: electroconvulsive therapy is only for depression refractory to multiple meds or depression with psychotic features • Choice C: buproprion is CI in this patient due to her history of bulimia nervosa (increased seizure risk) • Choice D: nortriptyline is a TCA; poor tolerability and severe adverse effects

90) A 5-year-old girl is brought to the office for evaluation of night awakenings. The patient has awakened screaming once or twice each month over the past few months shortly after going to sleep. During these episodes, she is short of breath, crying, sweating, and her face is flushed. The patient appears frightened, does not respond to her parents' attempts at comforting her, and continues to cry. She eventually goes back to sleep on her own and does not recall the incident the next morning. Until starting kindergarten 2 months ago, the patient stayed at home with her mother, and she no longer naps during the day. Vital signs and physical exam are unremarkable. Which of the following is the next best step in the management of this patient? a. Cognitive behavioral therapy b. Daytime naps c. Intranasal desmopressin d. Polysomnography e. Reassurance

E 90. Diagnosis = sleep terrors (NREM sleep, inconsolable, cannot be fully awakened, no memory of the incident); treatment = reassurance (clinical diagnosis; sleep terrors are not dangerous and resolve spontaneously in 1-2 years)

96) A 20-year-old woman comes to the office at her parents' urging. She is a college student required to give presentations in front of the class, during which she experiences severe anxiety. The patient says, "My face turns bright red, my hands start shaking, and I break out in a sweat. I had to leave the classroom in the middle of my last presentation." The patient is worried as she has several important presentations coming up in the next few weeks. The patient enjoys socializing with her friends at parties. Mental status exam shows an initially anxious patient who relaxes as the interview progresses. Physical exam is unremarkable. Which of the following is the most appropriate pharmacotherapy for this patient? a. Buproprion b. Buspirone c. Lorazepam d. Paroxetine e. Propanolol

E 96. Diagnosis = social anxiety disorder, performance-only; treatment = beta blocker PRN

97) A 21-year-old woman comes to the office for a routine exam. The patient shares that looking in the mirror has been increasingly difficult due to feeling upset about "how my breasts look." The patient frequently imagines what it would be like to not have breasts and wears a chest binder to make them appear flatter. The patient says, "I've always wanted to change how I look. I don't want people to see me as a woman anymore because I've never felt like one." The patient appears somewhat anxious and is wearing trousers and a loose T-shirt. Which of the following is the most likely explanation of this patient's condition? a. Adjustment disorder b. Body dysmorphic disorder c. Dissociative identity disorder d. Fetishistic disorder e. Gender dysphoria

E 97. Features of gender dysphoria: • Persistent (≥ 6 months) inconcgruence between assigned and felt gender • Dislikes own anatomy; wants to change body • Causes significant distress/impairment

54) A 26-year-old graduate student comes to the campus medical clinic because she was raped after a date 5 weeks ago. The patient tearfully describes the incident and acknowledges that she did not report it to police because she was concerned that her parents would pull her out of college. Since the assault, she has had recurring nightmares of the rape and takes the long way to class so that she does not walk past where the rape occurred. Occasionally, she hears her assailant's voice or feels his breath on her face. It takes the patient almost an hour to fall asleep, and she is afraid to be alone. In addition, she has extreme difficulty concentrating in class and would rather not attend at all despite previously enjoying school. This patient is at greatest risk for developing which of the following? a. Avoidant personality disorder b. Borderline personality disorder c. Obsessive compulsive disorder d. Schizophrenia e. Suicidal ideation and attempts

E Diagnosis = PTSD; sexual assault victims = increased lifetime risk for major depression and suicidal ideation/attempts


Kaugnay na mga set ng pag-aaral

Sadlier Vocabulary Workshop - Level B - Unit 6 - Antonyms

View Set

Life and health insurance - exam fx part 5

View Set

Australian States, Territories and Capital Cities

View Set

Underground Railroad and Slavery

View Set

subsaharan africa, southwest asia, north africa

View Set